- PowerScore Staff
- Posts: 5972
- Joined: Mar 25, 2011
- Fri Mar 16, 2012 2:47 pm
#85538
Complete Question Explanation
(The complete setup for this game can be found here: lsat/viewtopic.php?f=307&t=9348)
The correct answer choice is (B)
The question asks for a pair of buildings that can be inspected together only on Monday. G is a natural building choice for analysis since G is so restricted: G cannot be inspected on Wednesday, and if G is inspected on Tuesday, it would have to be inspected on Tuesday morning, forcing J to be inspected on Tuesday afternoon. Thus, on Tuesday, G must be inspected with J. Of course, at this point, you may be asking, “Why are we talking about Tuesday? Isn’t this question about Monday?” Yes, it is, but consider G’s situation at this juncture: it cannot be inspected on Wednesday, and on Tuesday it must be paired with J. Thus, if G is to be paired with any other variable besides J, that pairing must occur on Monday. Therefore, any answer choice pairing G and L, V, or Z would be correct. Only answer choice (B) makes such a pairing.
Note that the hypothetical created in question #13 answer choice (D) can be used to eliminate answer choice (A) from contention.
(The complete setup for this game can be found here: lsat/viewtopic.php?f=307&t=9348)
The correct answer choice is (B)
The question asks for a pair of buildings that can be inspected together only on Monday. G is a natural building choice for analysis since G is so restricted: G cannot be inspected on Wednesday, and if G is inspected on Tuesday, it would have to be inspected on Tuesday morning, forcing J to be inspected on Tuesday afternoon. Thus, on Tuesday, G must be inspected with J. Of course, at this point, you may be asking, “Why are we talking about Tuesday? Isn’t this question about Monday?” Yes, it is, but consider G’s situation at this juncture: it cannot be inspected on Wednesday, and on Tuesday it must be paired with J. Thus, if G is to be paired with any other variable besides J, that pairing must occur on Monday. Therefore, any answer choice pairing G and L, V, or Z would be correct. Only answer choice (B) makes such a pairing.
Note that the hypothetical created in question #13 answer choice (D) can be used to eliminate answer choice (A) from contention.
Dave Killoran
PowerScore Test Preparation
Follow me on X/Twitter at http://twitter.com/DaveKilloran
My LSAT Articles: http://blog.powerscore.com/lsat/author/dave-killoran
PowerScore Podcast: http://www.powerscore.com/lsat/podcast/
PowerScore Test Preparation
Follow me on X/Twitter at http://twitter.com/DaveKilloran
My LSAT Articles: http://blog.powerscore.com/lsat/author/dave-killoran
PowerScore Podcast: http://www.powerscore.com/lsat/podcast/